3

I'm currently working on the integral $I = \int_0^1 \sqrt\frac{1-x}{x}dx$. I would like to evaluate it with the dogbone contour specifically. Here's what I did so far:

Denote $f(z) = \sqrt\frac{1-z}{z}$ the integrand function (with complex variable). Then I observe that $$f(z) = (1 - z)^{\frac{1}{2}}z^{-\frac{1}{2}} = \frac{|1 - z|^{\frac{1}{2}}}{|z|^{\frac{1}{2}}}e^{\frac{1}{2}i(\arg(z) - \arg(1 - z))} (*)$$

Since $\ln(z) = \ln(|z|) + i\arg(z)$ (and similarly $\ln(1 - z) = \ln(|1 - z|) + i\arg(1 - z)$). Denote $Arg = \frac{1}{2}(\arg(z) - \arg(1 - z))$ for the step to come.

Next I defined $- \pi < \arg(z) \leq \pi$ and $0 \leq \arg(z) < 2\pi$, to observe the behaviour of Arg in the intervals $(- \infty, 0)$ and $(0,1)$.

  • In $(- \infty, 0)$, from above, we have: $\arg(z) \to \pi$, $\arg(1 - z) \to 2\pi \implies Arg \to \frac{1}{2}(\pi - 2\pi) = - \frac{\pi}{2}$

  • In $(- \infty, 0)$, from below, we have: $\arg(z) \to - \pi$, $\arg(1 - z) \to 0 \implies Arg \to \frac{1}{2}(- \pi - 0) = - \frac{\pi}{2}$

Hence we deduce that our function is continuous everywhere on this interval. However:

  • In $(0,1)$, from above, we have: $\arg(z) \to 0$, $\arg(1 - z) \to 2\pi \implies Arg \to \frac{1}{2}(0 - 2\pi) = - \pi$

  • In $(0,1)$, from below, we have: $\arg(z) \to 0$, $\arg(1 - z) \to 0 \implies Arg \to 0$

And in this interval our function is discontinuous since $e^{- i\pi} \ne e^0 = 1$. Thus I introduced a dogbone contour, namely $C$, aroud the branch $[0,1]$ such that:

enter image description here

$$C = \psi_1 \cup \psi_2 \cup c_0 \cup c_1$$

Where $\psi_1$ is the blue segment from $1$ to $0$ and $\psi_2$ is the blue segment from $0$ to $1$. (Thanks to @Zaid Alyafeai for the picture).

Then we have the equality: $\oint_C f(z)dz = \int_{\psi_1} f(z)dz + \int_{\psi_2} f(z)dz + \int_{c_1} f(z)dz + \int_{c_2} f(z)dz$; and pumping up $C$ we also have that $\oint_C = - 2\pi i (res_{z = 0}f(z)dz + res_{z = \infty}f(z)dz)$ by Cauchy Residue Theorem since $0,\infty$ are the only singular points of $f(z)$.

Then I computed them:

For $\int_{\psi_1} f(z)dz$ and $\int_{\psi_2} f(z)dz$, I used $z = t + i\epsilon$ with $t \in [0,1] $and $dz = dt$:

  • $$\int_{\psi_1}f(z)dz = - \int_0^1\frac{(1 - t - i\epsilon)^{\frac{1}{2}}}{(t + i\epsilon)^{\frac{1}{2}}}dt = - \int_0^1 \frac{|1 - t - i\epsilon|^{\frac{1}{2}}}{|t + i\epsilon|^{\frac{1}{2}}}e^{i\pi}dt$$ (I deduced the last integral from $(*)$ and from the arguments I computed above)

Then taking the limt when $\epsilon \to 0$ I finally obtained that $\lim\limits_{\epsilon \to 0} \int_{\psi_1}f(z)dz = I$.

  • In a very similar way I found that $\lim\limits_{\epsilon \to 0} \int_{\psi_2}f(z)dz = I$ as well.

Now for $\int_{c_0} f(z)dz$ and $\int_{c_1} f(z)dz$, I used $z = \epsilon e^{i\theta}$ with $\theta \in [0,2\pi]$ and $dz = i\epsilon e^{i\theta}$, and my goal was to find upper bounds of the absolute value of these integrals:

  • $|\int_{c_0} f(z)dz| = |i\epsilon^{\frac{1}{2}}\int_0^{2\pi}(1 - \epsilon e^{i\theta})^{\frac{1}{2}}e^{\frac{1}{2}i\theta}dt| \leq i\epsilon^{\frac{1}{2}}\int_0^{2\pi}|1 - \epsilon e^{i\theta}|^{\frac{1}{2}}|e^{\frac{1}{2}i\theta}|dt \leq i\epsilon^{\frac{1}{2}}(1 + \epsilon)^{\frac{1}{2}}2\pi$

Thus I had: $0 \leq \lim\limits_{\epsilon \to 0}|\int_{c_0}f(z)dz[ \leq 0$ which implies that $\lim\limits_{\epsilon \to 0}\int_{c_0}f(z)dz = 0$.

In a very similar way again, I found that $\lim\limits_{\epsilon \to 0}\int_{c_1}f(z)dz = 0$.

Hence in the end I had: $\oint_Cf(z)dz = I + I + 0 + 0 = 2I$, and since I also had that $\oint_Cf(z)dz = - 2\pi i (res_{z = 0}f(z)dz + res_{z = \infty}f(z)dz)$, it follows that:

$$2I = - 2\pi i (res_{z = 0}f(z)dz + res_{z = \infty}f(z)dz) (***)$$

And that's where my problem starts (or not !): I computed both residues and found they were both equal to 0, but this implies that $I = 0$ and I feel like this is wrong, because first I don't understand why they would be 0 when looking at their shape, and second this means the "number" of $I$ on the LHS of $(***)$ does not influence our result. So I hope you can help me either finding my mistake, either to show me that those residues are not 0 because I did not achieve to find another result (either to prove me that $I = 0$ because I don't think so).

I already appreciated your time, but any help would be welcome!

Rhaena
  • 361
  • 1
    Why not let $x=\sin ^2 \theta$? – Zerox May 04 '21 at 13:50
  • At which step do you suggest to use this change of variable @Zerox ? – Rhaena May 04 '21 at 14:28
  • Observe that $f^{-1}(x)=1/(1+x^2)$ and use $$\int f(x),dx=x\cdot f(x)-F^{-1}(f(x)).$$ – Michael Hoppe May 04 '21 at 19:35
  • I am interested by a way to draw such nice integration contours. I see you thank @Zaid Alyafeai for this one. Does it means that he has drawn it for you or he has provided an accessible tool for doing whatever circuit one needs ? – Jean Marie May 04 '21 at 20:33
  • I thanked him because I found this picture on google but noticed it came from a post on mathstack; here is the link of his post so you can ask him yourself: https://math.stackexchange.com/questions/2143137/integrating-around-a-dog-bone-contour. However if you find how to do this I would be very interested as well! – Rhaena May 05 '21 at 07:05
  • @Rhaena Thank you. I have a link to a tikz code for doing such contours but it's always better to see different ways for doing them. – Jean Marie May 05 '21 at 14:47

3 Answers3

2

$\DeclareMathOperator*{\res}{Res}$

I see two errors in your derivation. First you try to sum the residues at $0$ and $\infty$. This is not correct because they are in different parts of the complex plane created by your contour.

We need in fact only the residue at infinity and this appears to be not zero. The simplest way to find the residue is to expand the function into Laurent series in the annulus $|z|>1$ where the function is analytic: $$ f(z)=\sqrt\frac{1-z}z=i\sqrt{1-\frac1z}=i\left(1-\color{red}{\frac12}\frac1z-\frac18\frac1{z^2}-\cdots\right), $$ so that $$ I=\frac12\oint_C f(z)dz=-\pi i \res_{z=\infty} f(z)=-\pi i\left(\frac i2\right)=\frac\pi2. $$


The above value can be easily verified by real integration using substitution $x=\sin^2\theta$: $$ \int_0^1\sqrt\frac{1-x}xdx=2\int_0^{\pi/2}\cos^2\theta\,d\theta=\frac\pi2. $$

user
  • 26,272
  • Indeed I had not noticed that for $0$ and $\infty$ but I agree! Thank you very much for your help! – Rhaena May 04 '21 at 16:38
  • 1
    You're welcome! And thank you for the interesting question! – user May 04 '21 at 18:11
  • The residue at infinity is $-c_{-1}$ in the expansion around infinity. So for the branch of $f(z) = \sqrt {(1 - z)/z}$ that you chose, $\operatorname {Res}_{z = \infty} f(z) = i/2$. You need to consider $f(x \pm i0)$ to show how the residue is related to the integral over $[0, 1]$. And, strictly speaking, since your Laurent series converges on $D = {z: |z| > 1 }$, you need to expand the contour $[-i0, 1 - i0] \cup [1 + i0, i0]$ to a contour lying in $D$. – Maxim May 05 '21 at 12:10
  • It is not explained though why the sign of the residue has changed but the integral over $[0, 1]$ hasn't, or, in other words, why choosing $-f(z)$ wouldn't give $-\pi/2$ for the integral of the positive square root over $[0, 1]$ (which is why we need to consider the sign of $f(x + i0)$). And yes, I think it needs to be clarified that we can either expand the contour and integrate the Laurent series or just apply the residue theorem because there is a branch analytic on $\mathbb C \setminus [0, 1]$, not just on $D$. – Maxim May 05 '21 at 12:50
  • @Maxim Thank you once more for the profound and useful comments. I have realized that my approach was confusing and in fact I calculated the residue at $0$, not at $\infty$. Thus, the signs of the residue and the resulting integral are correct. What concerns your suggestion to consider $f(x\pm i0)$ it is of course absolutely correct but as far as I can see this was done already in OP (deriving the expression $2I=\int_C f(z),dz$). – user May 05 '21 at 15:45
  • The residue at $0$ is undefined, $f$ is not analytic in a punctured neighborhood of $0$. If we start by fixing an analytic branch on $\mathbb C \setminus [0, 1]$ by the condition $f(x + i0) < 0$ for $0 < x < 1$ (so that $I = -\int_{i0}^{1 + i0} f(z) dz$), we then have to show that the Laurent series agrees with this $f$ (not $-f$). There's nothing wrong with computing the residue at infinity, I was just pointing out that, unless we simply ignore the sign because we know it in advance, we need to show the relation between $I$, $\int_{i0}^{1 + i0} f(z) dz$ and the Laurent series. – Maxim May 05 '21 at 16:37
1

Are you required to use contour integration? Because there is an easier way. Let $$s=\sqrt{\frac{1-x}{x}}\iff x=\frac{1}{s^2+1}\implies \mathrm{d}s=\frac{-2u}{(1+s^2)^2}~\mathrm{d}s$$ So $$\int_0^1 \sqrt{\frac{1-x}{x}}\mathrm{d}x=\int_{\infty}^0s~\frac{-2s}{(1+s^2)^2}\mathrm{d}s$$ $$=\int_0^\infty \frac{2s^2}{(1+s^2)^2}\mathrm{d}s$$ Now you can integrate by parts. Let $$u=s,\mathrm{d}u=\mathrm{d}s$$ $$\mathrm{d}v=\frac{2s}{(1+s^2)^2}\mathrm{d}s\implies v=\frac{-1}{s^2+1}$$ Where the last bit follows from a simple substitution $t=s^2$. So, $$\int_0^\infty u~\mathrm{d}v=(uv)\big|^{s=\infty}_{s=0}-\int_0^\infty v~\mathrm du$$ $$=\left(\frac{-s}{s^2+1}\right)\bigg|^\infty_0+\int_0^\infty \frac{1}{1+s^2}\mathrm{d}s$$ And I'm sure you know the last integral by heart! The first term vanishes and we are left simply with $$\int_0^1\sqrt{\frac{1-x}{x}}\mathrm{d}x=\int_0^\infty \frac{1}{1+s^2}\mathrm{d}s=\frac{\pi}{2}$$

K.defaoite
  • 12,536
  • I am actually, but thank you very much for your integration, it proves my idea that $I \ne 0$ ! So sorry for not saying it clearly enough, I shall edit that right away – Rhaena May 04 '21 at 15:22
0

An easier way is: \begin{align} \int_0^1\sqrt{\dfrac{1-x}{x}}dx&=\frac12\int_0^1\frac{2-2x}{\sqrt{x(1-x)}}dx\\&=\frac12\left(\int_0^1\frac{1-2x}{\sqrt{x-x^2}}dx+\int_0^1\frac{1}{\sqrt{(\frac12)^2-(x-\frac12)^2}}dx\right)\\ &=\frac12\left(2\sqrt{x-x^2}\Bigg|_0^1+\left.\sin^{-1}\left(\frac{x-\frac12}{\frac12}\right)\right|_0^1\right)\\ &=\boxed{\frac\pi2} \end{align}

Martund
  • 14,706
  • 2
  • 13
  • 30
  • It is indeed ! But I would like to use dogbone contour specifically.. – Rhaena May 04 '21 at 15:25
  • Even easier way is to use $x=\sin^2\theta$. – user May 04 '21 at 15:33
  • @user, "easier" or "more difficult" are subjective terms, and hence personal choices. You may find that easier, but I personally find this solution much more easier than any other way. – Martund May 04 '21 at 15:47